Đến nội dung

Hình ảnh

[Chuyên đề] Một số vấn đề về bất đẳng thức bậc bốn

- - - - -

  • Please log in to reply
Chủ đề này có 12 trả lời

#1
Nguyenhuyen_AG

Nguyenhuyen_AG

    Trung úy

  • Thành viên nổi bật 2016
  • 945 Bài viết

*
Phổ biến

Đây là một bài viết nhỏ của mình và một bạn bên Y Dược, nội dung chủ yếu của bài viết xoay quanh một vài công cụ và mẹo nhỏ để giải quyết các bất đẳng thức bậc bốn ba biến số. Bài này mình viết cho kỷ yếu GGTH 2015 bạn nào có đi GGTH chắc sẽ thấy. Mình xin được chia sẽ cho các bạn không có dịp được đi GGTH.

1. Mở đầu

 

Trên tạp chí IJPAM (International Journal of Pure and Applied Mathematics) nhóm tác giả Vasile Cirtoaje (Department of Automatic Control and Computers, Ploiesti, Romania) và Võ Quốc Bá Cẩn (Archimedes Academy, Hà Nội) có đề xuất định lý :

Với $a,\,b,\,c$ là ba số thực, xét đa thức

\[F(a,b,c) = \sum a^4 + A \sum b^2c^2 + B abc \sum a + C \sum b^3c + D\sum bc^3.\]

Nếu $1+A+B+C+D=0,$ bất đẳng thức $F(a,b,c)\ge0$ sẽ đúng khi và chỉ khi

\[3(1+A) \ge C^2+CD+D^2.\]
Có thể nói với định lý này lớp các bài toán bậc $4$ ba biến số đã được giải quyết triệt để. Tuy nhiên việc chứng minh và áp dụng nó trong các kỳ thi không phải là điều dễ dàng. Theo quan điểm cá nhân định lý này giống như một tiêu chuẩn trong việc sáng tạo ra các bất đẳng thức mới hơn là một phương pháp để giải quyết các bài toán. Điều mà chúng ta quan tâm không chỉ là những định lý tổng quát mà còn là những công cụ không quá quá phức tạp để sử dụng trong phòng thi. Vì thế, trong bài viết nhỏ này, chúng tôi xin chia sẻ một vài kỹ thuật nhỏ khá hiệu quả trong việc xử lý các bất đẳng thức bậc $4$ ba biến.

 

2. Các bài toán đối xứng

Năm $2011$ trên diễn đàn toán học artofproblemsolving.com thành viên mudok có đề xuất bài toán sau đây :

Bài 2.1. Cho các số thực $a,\,b,\,c$ thỏa mãn $a+b+c=0.$ Chứng minh rằng
\[(ab+bc+ca)^2+9abc\ge 3(ab+bc+ca). \quad (2.1.1)\]

Lời giải. Cách tự nhiên nhất là ta sẽ rút $c=-a-b$ rồi thay vào $(2.1.1),$ và được
\[\left [ ab-(a+b)^2\right ]^2-9ab(a+b)\ge 3\left [ab-(a+b)^2  \right ],\]
\[(a^2+ab+b^2)^2+3(a^2+ab+b^2) \ge 9ab(a+b).\]
Quan sát một chút ta thấy bất đẳng thức lúc này chứa ba đại lượng $a^2+ab+b^2,\,ab,\,a+b$ và chúng có mối liên hệ với nhau thông qua đánh giá
\[a^2+ab+b^2 \ge \frac{3}{4}(a+b)^2 \ge 3ab,\]
suy ra
\[(a^2+ab+b^2)^2+3(a^2+ab+b^2) \ge \frac{9}{16}(a+b)^4+9ab.\]
Như vậy ta chỉ cần chứng minh
\[\frac{(a+b)^4}{16} + ab \ge 9ab(a+b).\]
Rất tiếc bất đẳng thức này không đúng (có thể kiểm tra với $a=-1,\,b=1$). Do đó ta sẽ đổi lại đánh giá theo kiểu
\[\begin{aligned}(a^2+ab+b^2)^2+3(a^2+ab+b^2) &\ge (3ab)^2+3\cdot\frac{3}{4}(a+b)^2 \\& = 9a^2b^2+\frac{9}{4}(a+b)^2,\end{aligned} \quad (2.1.2)\]
và quy bài toán về chứng minh
\[a^2b^2+\frac{(a+b)^2}{4} \ge ab(a+b).\]
Bất đẳng thức này đúng theo bất đẳng thức AM-GM, nhưng có một lỗi trong đánh giá $(2.1.2)$ đó là $(a^2+ab+b^2)^2\ge (3ab)^2$ chỉ đúng khi $ab \ge 0,$ nhưng ta có thể khắc phục lỗi này bằng kỹ thuật sau.

Do $ab \cdot bc \cdot ca = a^2b^2c^2 \ge 0$ nên trong ba số $ab,\,bc,\,ca$ sẽ có ít nhất một số không âm, giả sử $ab \ge 0.$ Với những lập luận trên, áp dụng bất đẳng thức AM-GM, ta có
\[a^2+ab+b^2 \ge \frac{3}{4}(a+b)^2\ge 3ab \ge 0. \quad (2.1.3)\]
Suy ra
\[(a^2+ab+b^2)^2+3(a^2+ab+b^2) \ge 9a^2b^2+\frac{9(a+b)^2}{4}.\]
Ta sẽ chứng minh
\[a^2b^2+\frac{(a+b)^2}{4} \ge ab(a+b).\]
Cũng theo bất đẳng thức AM-GM thì
\[a^2b^2+\frac{(a+b)^2}{4} \ge 2\sqrt{a^2b^2\cdot\frac{(a+b)^2}{4}}= ab\left | a+b \right | \ge ab(a+b).\quad (2.1.4)\]
Đẳng thức xảy ra khi $(2.1.3)$ và $(2.1.4)$ trở thành đẳng thức, tức
\[\left\{ \begin{aligned}
& a = b \\
& a^2b^2 = \frac{(a+b)^2}{4} \end{aligned}\right.\]
Giải hệ này ta được $a = b = c =0$ và $a=b=1,\, c=-2.$ Bài được chứng minh.

Bài 2.2. Cho ba số thực $a,\,b,\,c$ thỏa mãn $a+b+c=3.$ Chứng minh rằng
\[3(a^4+b^4+c^4)+a^2+b^2+c^2+6\ge 6(a^3+b^3+c^3). \quad (2.2.1)\]

Lời giải. Đây cũng là một bài toán đối xứng, nên ta sẽ tìm cách đưa điều kiện về dạng tổng bằng $0,$ rồi dùng phép thế như trên. Rất tự nhiên đặt $a=x+1,\,b=y+1,\,c=z+1,$ khi đó
\[x+y+z= (a-1) + (b-1) + (c-1) = 0.\]
Với phép đặt này thì
\[\begin{aligned} a^2+b^2+c^2&=(x+1)^2+(y+1)^2+(z+1)^2\\&=x^2+y^2+z^2+2(x+y+z)+3\\&=x^2+y^2+z^2+3, \end{aligned}\]
tương tự
\[\begin{aligned}
a^3+b^3+c^3& = x^3+y^3+z^3+3(x^2+y^2+z^2)+3, \\
a^4+b^4+c^4& =x^4+y^4+z^4+4(x^3+y^3+z^3)+6(x^2+y^2+z^2)+3.
\end{aligned}\]
Bất đẳng thức $(2.2.1)$ trở thành
\[3(x^4+y^4+z^4)+6(x^3+y^3+z^3)+x^2+y^2+z^2 \ge 0. \quad (2.2.2)\]
Giả sử $xy \ge 0,$ rồi thay $z=-x-y$ vào $(2.2.2),$ ta được
\[3[x^4+y^4+(x+y)^4]+6[x^3+y^3-(x+y)^3]+x^2+y^2+(x+y)^2 \ge 0,\]
\[3(x^4+2x^3y+3x^2y^2+2xy^3+y^4)+x^2+xy+y^2\ge 9xy(x+y)\]
\[3(x^2+xy+y^2)^2+x^2+xy+y^2\ge 9xy(x+y).\]
Áp dụng bất đẳng thức AM-GM, ta có
\[x^2+xy+y^2 \ge \frac{3}{4}(x+y)^2 \ge 3xy \ge 0, \quad (2.2.3)\]
suy ra
\[3(x^2+xy+y^2)^2+x^2+xy+y^2\ge 27x^2y^2+\frac{3}{4}(x+y)^2.\]
Do đó ta chỉ cần chứng minh
\[9x^2y^2+\frac{(x+y)^2}{4} \ge 3xy(x+y).\]
Cũng theo bất đẳng thức AM-GM, thì
\[9x^2y^2+\frac{(x+y)^2}{4} \ge 2\sqrt{9x^2y^2\cdot\frac{(x+y)^2}{4}}=3xy\left | x+y \right |\ge 3xy(x+y). \quad (2.2.4)\]
Đẳng thức xảy ra khi $(2.2.3)$ và $(2.2.4)$ trở thành đẳng thức, tức $x,\,y$ là nghiệm của hệ
\[\left\{ \begin{aligned}
& x=y \\
& 27x^2y^2 = \frac{3}{4}(x+y)^2 \end{aligned}\right.\]
Giải hệ này ta được $x=y=0$ hoặc $x=y=\frac{1}{3},$ suy ra $a=b=c=1,$ hoặc $a=b=\frac{4}{3},\,c=\frac{1}{3}$ cùng các hoán vị. Bài toán được chứng minh.

 

Nhận xét. Đây là một bất đẳng thức khá thú vị, có rất nhiều lời giải và đơn giản nhất là dùng phương pháp dồn biến. Nhưng với kỹ thuật này chúng ta có một lời giải khá nhẹ nhàng, cơ bản.

 

Sau $2$ ví dụ đầu ta thấy ý tưởng chính để giải quyết các bất đẳng thức đối xứng bậc $4$ được “mô hình” hóa như sau:

 

  • Nếu bất đẳng thức có điều kiện $a+b+c=3$ thì ta sẽ tiến hành đổi biến sang $x=a-1,\,y=b-1,\,z=c-1,$ còn nếu bất đẳng thức có các điều kiện khác thì ta cố gắng thuần nhất bài toán hoặc thông qua một số đánh giá để đưa bài toán về đồng bậc sau đó chuẩn hóa $a+b+c=3,$ rồi sử dụng các biến đổi

\[\begin{aligned} a^4+b^4+c^4 &= x^4+y^4+z^4 + 4(x^3+y^3+z^3) + 6(x^2+y^2+z^2) + 3, \\ a^3+b^3+c^3 &= x^3+y^3+z^3 + 3(x^2+y^2+z^2) + 3, \\ a^2+b^2+c^2 &= x^2+y^2+z^2 + 3, \\ ab+bc+ca &= xy+yz+zx + 3, \\ abc &= xyz + xy+yz+zx + 1 \\ & \cdots \end{aligned}\]

  • Thay $z=-x-y$ rồi biến đổi về hai đại lượng ${{x}^{2}}+xy+{{y}^{2}},\, xy\left( x+y \right)$ thông qua các đẳng thức

\[\begin{aligned} x^4+y^4+z^4 &= 2(x^2+xy+y^2)^2, \\ x^3+y^3+z^3 &= -3xy(x+y),\\ x^2+y^2+z^2 &= 2(x^2+xy+y^2), \\ xy+yz+zx &= -(x^2+xy+y^2), \\ xyz &= -xy(x+y), \\ & \cdots\end{aligned}\]
Cuối cùng ta sẽ thu được bất đẳng thức có dạng
\[A{{({{x}^{2}}+xy+{{y}^{2}})}^{2}}+B({{x}^{2}}+xy+{{y}^{2}})+Cxy\left( x+y \right)+D\ge 0.\]
Từ đó tùy vào các hệ số các $A,\,B,\,C,\,D$ mà ta có những đánh giá thích hợp để giải quyết bài toán.

Rất vui mừng vì phát hiện thú vị này chúng tôi đã thử áp dụng  vào các bài toán khác và thu được những kết quả sau.

 

Bài 2.3. Cho ba số thực $a,\,b,\,c$ thỏa mãn $a+b+c=3.$ Chứng minh rằng
\[(ab+bc+ca-3)^2\ge 27(abc-1). \quad (2.3.1)\]

Lời giải. Đặt $a=x+1,\,b=y+1,\,c=z+1,$ thì $x+y+z=0.$ Khi đó
\[\begin{aligned}
ab+bc+ca &= xy+yz+zx+3. \\
abc& = xyz+xy+yz+xz+1.
\end{aligned}\]
Bất đẳng thức $(2.3.1)$ trở thành
\[(xy+yz+zx)^2\ge 27(xyz+xy+yz+xz). \quad (2.3.2)\]
Giả sử $xy \ge0,$ thay $z=-x-y$ vào $(2.3.2)$ và thu gọn lại, ta được
\[[xy-(x+y)^2]^2\ge 27[-xy(x+y)+xy-(x+y)^2],\]
hay là
\[(x^2+xy+y^2)^2+27(x^2+xy+y^2)+27xy(x+y) \ge 0.\]
Áp dụng bất đẳng thức AM-GM, ta có
\[x^2+xy+y^2 \ge \frac{3}{4}(x+y)^2\ge 3xy \ge 0,\]
suy ra
\[(x^2+xy+y^2)^2+27(x^2+xy+y^2) \ge 9x^2y^2+\frac{81}{4}(x+y)^2.\]
Ta sẽ chứng minh
\[x^2y^2+\frac{9}{4}(x+y)^2+3xy(x+y) \ge 0,\]
bất đẳng thức này đúng vì
\[x^2y^2+\frac{9}{4}(x+y)^2+3xy(x+y) = \frac{(3x+3y+2xy)^2}{4} \ge 0.\]
Đẳng thức xảy ra khi $a=b=c=1,$ hoặc $a=b=-2,\, c=7$ cùng các hoán vị. Chứng minh của ta được hoàn tất.

Bài 2.4. Cho ba số thực $a,\,b,\,c$ thỏa mãn $a+b+c=3.$ Chứng minh rằng
\[3(a^4+b^4+c^4)+33 \ge 14(a^2+b^2+c^2). \quad (2.4.1)\]

Lời giải. Đặt $a=x+1,\,b=y+1,\,c=z+1,$ thì $x+y+z=0.$ Bất đẳng thức $(2.4.1)$ được viết lại như sau
\[3(x^4+y^4+z^4) +12(x^3+y^3+z^3)+4(x^2+y^2+z^2) \ge 0. \quad (2.4.2)\]
Giả sử $xy \ge0,$ thay $z=-x-y$ vào $(2.4.2)$ và thu gọn lại, ta được
\[3(x^4+2x^3y+3x^2y^2+2xy^3+y^4)+4(x^2+xy+y^2) \ge 18xy(x+y),\]
\[3(x^2+xy+y^2)^2+4(x^2+xy+y^2) \ge 18xy(x+y).\]
Áp dụng bất đẳng thức AM-GM, ta có
\[x^2+xy+y^2 \ge \frac{3}{4}(x+y)^2\ge 3xy \ge 0,\]
suy ra
\[3(x^2+xy+y^2)^2+4(x^2+xy+y^2) \ge 27x^2y^2+3(x+y)^2,\]
lại có
\[27x^2y^2+3(x+y)^2-18xy(x+y)=3(x+y-3xy)^2 \ge 0.\]
Đẳng thức xảy ra khi $a=b=c=1$ hoặc $a=b=\frac{5}{3},\, c=-\frac{1}{3}.$ Bài toán được chứng minh.

Bài 2.5. Cho ba số thực $a,\,b,\,c$ thỏa mãn $a+b+c=3.$ Chứng minh rằng
\[(ab+bc+ca)^2 +9\ge 18abc. \quad (2.5.1)\]

Lời giải. Đặt $a=x+1,\,b=y+1,\,c=z+1,$ thì $x+y+z=0.$ Khi đó bất đẳng thức $(2.5.1)$ trở thành
\[(xy+yz+zx+3)^2 +9\ge 18(1+xy+yz+zx+xyz),\]
hay
\[(xy+yz+zx)^2 \ge 12(xy+yz+zx)+18xyz. \quad (2.5.2)\]
Giả sử $xy \ge 0,$ thay $z=-x-y$ vào $(2.5.2),$ ta được
\[(x^2+xy+y^2)^2+12(x^2+xy+y^2) + 18xy(x+y) \ge 0.\]
Áp dụng bất đẳng thức AM-GM, ta có
\[x^2+xy+y^2 \ge \frac{3}{4}(x+y)^2\ge 3xy \ge 0,\]
suy ra
\[(x^2+xy+y^2)^2+12(x^2+xy+y^2) \ge 9x^2y^2+9(x+y)^2,\]
như vậy ta cần chỉ ra
\[x^2y^2+(x+y)^2 + 2xy(x+y) \ge 0,\]
tương đương với
\[(x+y+xy)^2 \ge 0.\]
Đẳng thức xảy ra khi $a=b=c=1$ hoặc $a=b=-1,\, c=5.$ Bài toán được chứng minh.

Bài 2.6. Chứng minh rằng bất đẳng thức
\[(a^2+b^2+c^2-2ab-2bc-2ca)^2+9(ab+bc+ca)^2\ge 30abc(a+b+c), \quad (2.6.1)\]
luôn đúng với mọi số thực dương $a,\,b,\,c.$

Lời giải. Nếu $a+b+c=0$ thì $(2.6.1)$ hiển nhiên đúng. Nếu $a+b+c \ne 0,$ thay $(a,\, b,\, c)$ bởi $(-a,\, -b,\, -c)$ thì bất đẳng thức vẫn không thay đổi nên ta có thể giả sử $a+b+c>0,$ và chuẩn hóa cho $a+b+c=3.$ Đặt $a=x+1,\,b=y+1,\,c=z+1,$ khi đó $x+y+z=0.$ Bất đẳng thức $(2.6.1)$ trở thành
\[[4(xy+yz+zx)+3]^2 +9(xy+yz+zx+3)^2 \ge 90(xy+yz+zx+xyz),\]
tương đương với
\[25(xy+yz+zx)^2 \ge 12(xy+yz+zx)+90xyz. \quad (2.6.2)\]
Giả sử $xy \ge0,$ thay $z=-x-y$ vào $(2.6.2)$ và thu gọn lại, ta được
\[25(x^2+xy+y^2)^2+12(x^2+xy+y^2) + 90xy(x+y) \ge 0.\]
Áp dụng bất đẳng thức AM-GM, ta có
\[x^2+xy+y^2 \ge \frac{3}{4}(x+y)^2\ge 3xy \ge 0,\]
suy ra
\[ 25(x^2+xy+y^2)^2+12(x^2+xy+y^2) \ge 225x^2y^2 + 9(x+y)^2.\]
Ta sẽ chứng minh
\[25x^2y^2 + (x+y)^2 + 10xy(x+y) \ge 0,\]
điều này đúng vì
\[25x^2y^2 + (x+y)^2 + 10xy(x+y) = (x+y+5xy)^2 \ge 0.\]
Đẳng thức xảy ra khi $a=b=c$ hoặc $a=3b=3c$ cùng các hoán vị, như vậy chứng minh của ta được hoàn tất.

Bài 2.7. Cho ba số thực $a,\,b,\,c$ không âm. Chứng minh rằng
\[3(a^2+b^2+c^2) \ge (a+b+c)\left(\sqrt{ab}+\sqrt{bc}+\sqrt{ca}\right)+\sum (b-c)^2 \ge (a+b+c)^2.\]

(Việt Nam MO 2005)

Lời giải.Thay $(a,b,c)$ bởi $(a^2,b^2,c^2)$ bất đẳng thức cần chứng minh trở thành
\[3(a^4+b^4+c^4) \geqslant (a^2+b^2+c^2)(ab+bc+ca)+\sum (b^2-c^2)^2 \geqslant (a^2+b^2+c^2)^2.\]
Ta chứng minh vế trái
\[3(a^4+b^4+c^4) \geqslant (a^2+b^2+c^2)(ab+bc+ca)+2(a^4+b^4+c^4-a^2b^2-b^2c^2-c^2a^2).\]
Điều này tương đương với
\[a^4+b^4+c^4 +2(a^2b^2+b^2c^2+c^2a^2)\geqslant (a^2+b^2+c^2)(ab+bc+ca),\]
\[(a^2+b^2+c^2)^2\geqslant (a^2+b^2+c^2)(ab+bc+ca). \quad (2.7.1)\]
Chuẩn hóa $a+b+c=3$ và đặt $a=x+1,\,b=y+1,\,c=-x-y+1$ vào $(2.7.1)$ và thu gọn lại, ta được
\[6(x^4+2x^3y+3x^2y^2+2xy^3+y^4)+9(x^2+xy+y^2) \ge 0,\]
\[6(x^2+xy+y^2)^2+9(x^2+xy+y^2) \ge 0.\]
Bất đẳng thức này hiển nhiên đúng. Đẳng thức xảy ra khi $a=b=c,$ vế trái được chứng minh. Tiếp đến ta chứng minh vế bên phải
\[(a^2+b^2+c^2)(ab+bc+ca)+2(a^4+b^4+c^4-a^2b^2-b^2c^2-c^2a^2) \geqslant (a^2+b^2+c^2)^2,\]
bất đẳng thức này tương đương
\[a^4+b^4+c^4+(a^2+b^2+c^2)(ab+bc+ca) \geqslant 4(a^2b^2+b^2c^2+c^2a^2),\]
\[a^4+b^4+c^4+abc(a+b+c)+ab(a^2+b^2)+ca(c^2+a^2)+ca(c^2+a^2)\geqslant 4(a^2b^2+b^2c^2+c^2a^2).\]
Dễ thấy này là hệ quả của hai bất đẳng thức sau đây
\[ab(a^2+b^2)+ca(c^2+a^2)+ca(c^2+a^2) \ge 2(a^2b^2+b^2c^2+c^2a^2), \quad (2.7.2)\]
\[a^4+b^4+c^4+abc(a+b+c) \ge ab(a^2+b^2)+ca(c^2+a^2)+ca(c^2+a^2). \quad (2.7.3)\]
Bất đẳng thức $(2.7.2)$ tương đương với
\[ab(a-b)^2+bc(b-c)^2+ca(c-a)^2 \ge 0.\]
Ta chứng minh $(2.7.3),$ chuẩn hóa cho $a+b+c=3$ và đặt $a=x+1,\,b=y+1,c=-x-y+1$ với $xy \ge 0$ rồi thế vào bất đẳng thức trên và thu gọn lại, ta được
\[4(x^2+xy+y^2)^2+3(x^2+xy+y^2) \ge 18xy(x+y).\]
Theo bất đẳng thức AM-GM ta có
\[\begin{aligned} 4(x^2+xy+y^2)^2+3(x^2+xy+y^2) &\ge 4\cdot (3xy)^2 +3 \cdot \frac{3}{4}(x+y)^2\\&=36x^2y^2+\frac{9}{4}(x+y)^2, \end{aligned}\]

\[36x^2y^2+\frac{9}{4}(x+y)^2-18xy(x+y) = \frac{9(x+y-4xy)^2}{4} \ge 0.\]
Đẳng thức xảy ra khi $a=b=c$ hoặc $a=b,\, c=0.$ Như vậy bài toán được chứng minh.

Bài 2.8. Cho ba số thực $a,\,b,\,c$ thỏa mãn $abc=-1,$ chứng minh rằng
\[a^4+b^4+c^4+3(a+b+c)\ge \frac{a^2+b^2}{c}+\frac{b^2+c^2}{a}+\frac{c^2+a^2}{b}.\]

(Iran MO 2005)

Lời giải. Ta viết bất đẳng thức lại dưới dạng thuần nhất như sau
\[a^4+b^4+c^4-3abc(a+b+c)\ge -ab(a^2+b^2)-bc(b^2+b^2)-ca(c^2+a^2). \quad (2.8.1)\]
Vì  
\[\sum ab(a^2+b^2)=(ab+bc+ca)(a^2+b^2+c^2)-abc(a+b+c),\]
nên $(2.8.1)$ tương đương với
\[a^4+b^4+c^4+(ab+bc+ca)(a^2+b^2+c^2)\ge 4abc(a+b+c). \quad (2.8.2)\]
Nếu $a+b+c=0,$ thay $c=-a-b$ vào $(2.8.2)$ ta được
\[a^4+b^4+(a+b)^4-(a^2+ab+b^2)\left[a^2+b^2+(a+b)^2\right] \ge 0,\]
tuy nhiên dễ thấy
\[a^4+b^4+(a+b)^4-(a^2+ab+b^2)\left[a^2+b^2+(a+b)^2\right] = 0,\]
nên bài toán đúng trong trường hợp này.

Nếu $a+b+c \ne 0,$ thay $(a,\, b,\, c)$ bởi $(-a,\, -b,\, -c)$ thì bất đẳng thức vẫn không thay đổi nên ta có thể giả sử $a+b+c>0,$ và chuẩn hóa cho $a+b+c=3.$

Đặt $a=x+1,\,b=y+1$ thì $c=-x-y+1,$ rồi thay vào $(2.8.2)$ và thu gọn lại ta được
\[27(x^2+xy+y^2) \ge 0.\]
Bất đẳng thức này luôn đúng. Đẳng thức xảy ra khi $a=b=c$ hoặc $a+b+c=0.$ Bài toán được chứng minh.

Nhận xét. Bài toán là hệ quả của đẳng thức sau
\[\sum a^4+\sum bc\sum a^2=(a+b+c)^2(a^2+b^2+c^2-ab-bc-ca)+4abc(a+b+c).\]

Bài 2.9. Chứng minh rằng với mọi số thực $a,\, b,\, c$ ta luôn có
\[(a+b)^4+(b+c)^4+(c+a)^4\ge \frac{4}{7}\left[a^4+b^4+c^4+(a+b+c)^4\right]. \quad (2.9.1)\]

Lời giải. Nếu $a+b+c=0,$ thay $c=-a-b$ vào $(2.9.1)$ và thu gọn ta được
\[a^4+2a^3b+3a^2b^2+2ab^3+b^4\ge 0,\]
ta có
\[a^4+2a^3b+3a^2b^2+2ab^3+b^4 = (a^2+ab+b^2)^2 \ge 0.\]
Nếu $a+b+c\ne 0$, thay $(a,\, b,\, c)$ bởi $(-a,\, -b,\, -c)$ thì bất đẳng thức vẫn không thay đổi nên ta có thể giả sử $a+b+c>0,$ và chuẩn hóa cho $a+b+c=3.$

Đặt $a=x+1,\,b=y+1,\,c=z+1,$ thì $x+y+z=0$ và $x^3+y^3+z^3=3xyz,$ ta có
$$(a+b)^4+(b+c)^4+(c+a)^4 = x^4+y^4+z^4+24(x^2+y^2+z^2)-24xyz+48,$$
tương tự thì
$$a^4+b^4+c^4+(a+b+c)^4 = x^4+y^4+z^4+6(x^2+y^2+z^2)+12xyz+84.$$
Như vậy bất đẳng thức trở thành
$$x^4+y^4+z^4+48(x^2+y^2+z^2)\ge 72xyz. \quad (2.9.2)$$
Giả sử $xy \ge 0$ rồi thay $z=-x-y$ vào $(2.9.2),$ bất đẳng thức được viết lại dưới dạng
\[2(x^2+xy+y^2)^2+96(x^2+xy+y^2)+72xy(x+y) \ge 0.\]
Theo bất đẳng thức AM-GM, ta có
\[2(x^2+xy+y^2)^2+96(x^2+xy+y^2) \ge 18x^2y^2+72(x+y)^2,\]
lại có
\[18x^2y^2+72(x+y)^2+72xy(x+y) = 18(2x+2y+xy)^2 \ge 0.\]

Đẳng thức xảy ra khi và chỉ khi $a=b=c$ hoặc $-3a=-3b=c.$ Chứng minh của chúng ta vì thế hoàn tất.

Nhận xét. Bài toán là một kết quả mạnh hơn của bất đẳng thức sau
\[(a+b)^4+(b+c)^4+(c+a)^4\ge \frac{4}{7}(a^4+b^4+c^4).\]

(Việt Nam TST 1996)

Bài 2.10. Với ba số thực $a,\, b,\, c.$ Chứng minh rằng với mọi số thực $k$ ta luôn có
\[(a-b)(a-c)(a-kb)(a-kc)+(b-c)(b-a)(b-kc)(b-ka)+(c-a)(c-b)(c-ka)(c-kb) \ge 0. \quad (2.10.1)\]
 
Lời giải. Lập luận như trên giả sử $a+b+c>0,$ và chuẩn hóa cho $a+b+c=3.$ Đặt $a=x+1,\,b=y+1,\,c=-x-y+1$ với $xy \ge 0,$ khi đó bất đẳng thức $(2.10.1)$ được viết lại dưới dạng
\[(k+2)^2(x^2+xy+y^2)^2+3(k-1)^2(x^2+xy+y^2) \ge 9(k+2)(1-k)xy(x+y). \quad (2.10.2)\]
Theo bất đẳng thức AM-GM, ta có
\[x^2+xy+y^2\ge \frac{3}{4}(x+y)^2 \ge 3xy \ge 0.\]
Suy ra
\[(k+2)^2(x^2+xy+y^2)^2+3(k-1)^2(x^2+xy+y^2) \ge 9(k+2)^2x^2y^2+\frac{9}{4}(k-1)^2(x+y)^2,\]
Ta sẽ chứng minh
\[(k+2)^2x^2y^2+\frac{(k-1)^2(x+y)^2}{4} \ge (k+2)(1-k)xy(x+y),\]
tương đương với
\[\left[(k-1)(x+y) + 2(k+2)xy\right]^2 \ge 0.\]
Đẳng thức xảy $a=b=c$ hoặc $a = kb = kc.$ Chứng minh hoàn tất.

Nhận xét. Bài toán là hệ quả của đẳng thức
\[2\sum (a-b)(a-c)(a-kb)(a-kc) = \sum (a-b)^2\left[ a + b - (k+1)c\right]^2.\]

Đây là một kết quả thú vị, chúng ta có một số kết quả khá đẹp mắt như sau

Nếu $k=0,$ bất đẳng thức trở thành
\[a^2(a-b)(a-c) + b^2(b-c)(b-a) + c^2(c-a)(c-b) \ge 0,\]
đây chính là bất đẳng thức Schur bậc 4 và nếu khai triển ta sẽ được bất đẳng thức $(2.7.3).$

Nếu $k=2,$ bất đẳng thức trở thành
\[9(a^4+b^4+c^4)+126(a^2b^2+b^2c^2+c^2a^2) \ge 5(a+b+c)^4.\]
Đẳng thức xảy ra khi $\frac{a}{2}=b=c.$


Bài viết đã được chỉnh sửa nội dung bởi Nguyenhuyen_AG: 26-08-2015 - 11:29

Nguyen Van Huyen
Ho Chi Minh City University Of Transport

#2
Nguyenhuyen_AG

Nguyenhuyen_AG

    Trung úy

  • Thành viên nổi bật 2016
  • 945 Bài viết

3. Đưa về bậc $4$

Có rất nhiều bài toán dạng phân thức, căn thức, ... thoạt nhìn sẽ không có dạng đa thức bậc $4$ nhưng nếu thông qua một số đánh giá và biến đổi, ta có thể chuyển chúng về dạng đa thức bậc $4$ và có thể áp dụng phương pháp trên. Một bất đẳng thức bậc $4$ kinh điển mà bạn đọc chắc hẳn đã ít nhất một lần được gặp đó là bất đẳng thức Schur bậc $4,$ trong phần này tác giả sẽ đưa các bài toán về bậc $4$ thông quá bất đẳng thức Schur. Để hiểu rõ hơn, mời ban đọc xét qua các bài toán sau.

 

Bài 3.1. Cho $a,\,b,\,c$ là các số thực không âm thỏa mãn $ab+bc+ca > 0.$ Chứng minh rằng bất đẳng thức sau luôn đúng
\[\frac{(a+b)^2}{c^2+ab}+\frac{(b+c)^2}{a^2+bc}+\frac{(c+a)^2}{b^2+ca} \ge 6.\]

Lời giải. Áp dụng bất đẳng thức Cauchy-Schwarz ta có
\[\begin{aligned} \sum\frac{(a+b)^2}{c^2+ab}&\ge \frac{[(a+b)^2+(b+c)^2+(c+a)^2]^2}{(ab+c^2)(a+b)^2+(bc+a^2)(b+c)^2+(ca+b^2)(c+a+b)^2}\\&=\frac{4(a^2+b^2+c^2+ab+bc+ca)^2}{(ab+c^2)(a+b)^2+(bc+a^2)(b+c)^2+(ca+b^2)(c+a+b)^2}.\end{aligned}\]
Vì thế để chứng minh bài toán ta cần chứng minh được
\[\frac{(a^2+b^2+c^2+ab+bc+ca)^2}{(ab+c^2)(a+b)^2+(bc+a^2)(b+c)^2+(ca+b^2)(c+a)^2}\ge \frac{3}{2}.\]
Bằng cách khai triển trực tiếp ta thấy điều này tương đương với
\[2[a^4+b^4+c^4+abc(a+b+c)-\sum ab(a^2+b^2)]+3\sum ab(a-b)^2 \ge 0.\]
Bất đẳng thức cuối cùng đúng theo bất đẳng thức $(2.7.3),$ đẳng thức xảy ra khi và chỉ khi $a=b=c$ hoặc $a=b,\,c=0$ và các hoán vị. Bài toán được chứng minh.

 

Bài 3.2. Cho $a,\,b,\,c$ là các số thực không âm thỏa mãn $a^2+b^2+c^2+ab+bc+ca=6.$ Chứng minh rằng
\[\frac{1}{4-ab}+\frac{1}{4-bc}+\frac{1}{4-ca} \le 1 \quad (3.2.1).\]

Lời giải. Bài toán này được làm chặt từ đề thi Moldova TST 2005 và có đến hai dấu bằng là $a=b=c=1$ và $a=b=\sqrt{2},\,c=0.$ Muốn sử dụng bất đẳng thức Cauchy-Schwarz ta phải dùng đến kỹ thuật thêm bớt để làm đảo chiều bài toán. Ta xét
\[P(a,b)=k-\frac{1}{4-ab}.\]
Ta có
\[\begin{aligned} P(1,1) &= k-\frac{1}{3}, \\
P\left(\sqrt{2},0\right) &= k-\frac{1}{4}, \\
P\left(\sqrt{2},\sqrt{2}\right) &= k-\frac{1}{2}.\end{aligned}\]
Ta cần tìm $k$ sao cho $P(a,b) \ge 0$ và đánh giá này càng chặt càng tốt từ các tính toán trên ta thấy $k \ge \frac{1}{2}$ là giá trị tốt nhất cần tìm, từ đó dẫn đến đẳng thức
\[1-\dfrac{2}{4-ab}=\dfrac{2-ab}{4-ab},\]
bất đẳng thức (3.2.1) được viết lại như sau
\[\frac{2-ab}{4-ab}+\frac{2-bc}{4-bc}+\frac{2-bc}{4-bc}\ge 1.\]
Do $a^2+b^2+c^2+ab+bc+ca=6,$ nên
\[3(2-ab) = (a-b)^2+c^2+bc+ca \ge 0.\]
Theo bất đẳng thức Cauchy-Schwarz, ta có
\[\sum \frac{2-ab}{4-ab} \ge \frac{\left [\sum (2-ab)  \right ]^2}{\sum (2-ab)(4-ab)}=\frac{(6-ab-bc-ca)^2}{\sum (2-ab)(4-ab)}=\frac{(a^2+b^2+c^2)^2}{\sum (2-ab)(4-ab)}.\]
Như vậy, ta cần chứng minh
\[(a^2+b^2+c^2)^2\ge \sum (2-ab)(4-ab),\]
tương đương với
\[a^4+b^4+c^4+a^2b^2+b^2c^2+c^2a^2+6(ab+bc+ca)\ge 24,\]
hay dưới dạng thuần nhất
\[\sum a^4+\sum a^2b^2+\left(\sum ab\right)\left ( \sum a^2+\sum bc \right )\ge \frac{2}{3}\left ( \sum a^2+\sum bc \right )^2.\]
Khi triển và thu gọn lại ta được
\[a^4+b^4+c^4+abc(a+b+c)\ge ab(a^2+b^2)+bc(b^2+c^2)+ca(c^2+a^2).\]
Đây chính là bất đẳng thức $(2.7.3).$ Bài toán được chứng minh.

 

Bài 3.3. Cho ba số thực $x,\,y,\,z$ thuộc $\left [ -1,1 \right ]$ thỏa mãn $x+y+z=0.$ Chứng minh rằng
\[\sqrt{1+x+y^2}+\sqrt{1+y+z^2}+\sqrt{1+z+x^2} \ge 3.\]

Lời giải. Bình phương hai vế, ta được
\[x^2+y^2+z^2 + 2\sum \sqrt{(1+x+y^2)(1+y+z^2)} \ge 6.\]
Áp dụng bất đẳng thức Cauchy-Schwarz ta có
\[\begin{aligned} \sum \sqrt{(1+x+y^2)(1+y+z^2)} &\ge \sum \sqrt{(1+x)(1+y)} + \sum \left | yz \right | \\& \ge \sum \sqrt{(1+x)(1+y)} - \sum yz.\end{aligned}\]
Ta đưa bài toán về chứng minh
\[\sum x^2 - 2\sum yz + 2\sum \sqrt{(1+x)(1+y)} \ge 6.\]
Đặt $1+x=a^2,\,1+y=b^2,\,1+z=c^2$ với $a,\,b,\,c$ là các số không âm, khi đó $a^2+b^2+c^2=3,$ bất đẳng thức trên trở thành
\[\sum a^4 - 2\sum b^2c^2 + 2\sum a^2 + 2\sum bc - 9 \ge 0,\]
tương đương với
\[\sum a^4 - 2\sum b^2c^2 + 2\sum bc - \sum a^2 \ge 0,\]
\[3\sum a^4 - 6\sum b^2c^2 + \left(2\sum bc - \sum a^2\right)\left(\sum a^2\right) \ge 0,\]
khai triển và thu gọn lại, ta được
\[a^4+b^4+c^4+abc(a+b+c)-\sum ab(a^2+b^2)+2\sum bc(b-c)^2 \ge 0.\]
Bất đẳng thức này đúng theo bất đẳng thức $(2.7.3),$ đẳng thức xảy ra khi $x=y=z=0.$ Bài toán được chứng minh.

 

Bài 3.4. Cho ba số thực không âm $a,\,b,\,c$ thỏa mãn $a^2+b^2+c^2>0.$ Chứng minh rằng
\[a^2+b^2+c^2+\sqrt{3}\cdot\frac{\sqrt[3]{abc}(ab+bc+ca)}{\sqrt{a^2+b^2+c^2}} \ge 2(ab+bc+ca).\]

Lời giải. Áp dụng bất đẳng thức AM-GM ta có
\[\sqrt{3}\cdot\frac{\sqrt[3]{abc}(ab+bc+ca)}{\sqrt{a^2+b^2+c^2}} \ge \frac{3\sqrt{3}abc}{\sqrt{a^2+b^2+c^2}}=\frac{3\sqrt{3}abc\sqrt{a^2+b^2+c^2}}{a^2+b^2+c^2} \ge \frac{3abc(a+b+c)}{a^2+b^2+c^2}.\]
Ta cần chứng minh
\[a^2+b^2+c^2+\frac{3abc(a+b+c)}{a^2+b^2+c^2}\ge 2(ab+bc+ca),\]
quy đồng và thu gọn lại ta được
\[a^4+b^4+c^4+abc(a+b+c)\ge ab(a^2+b^2)+bc(b^2+c^2)+ca(c^2+a^2).\]
Đây chính là bất đẳng thức $(2.7.3),$ đẳng thức xảy ra khi $a=b=c$ hoặc $a=b,\,c=0$ cùng các hoán vị. Chứng minh hoàn tất.

 

Bài 3.5. Cho ba số thực không âm $a,\,b,\,c$ thỏa mãn $a^3+b^3+c^3+abc=12.$ Chứng minh rằng bất đẳng thức sau luôn được thỏa mãn
\[\frac{19(a^2+b^2+c^2)+6(ab+bc+ca)}{a+b+c} \ge 36.\]

Lời giải. Áp dụng bất đẳng thức AM-GM, ta có
\[\begin{aligned} 19(a^2+b^2+c^2)+6(ab+bc+ca) &= 8(a^2+b^2+c^2)+8(a^2+b^2+c^2)+3(a+b+c)^2 \\& \ge 12\sqrt[3]{3(a^2+b^2+c^2)^2(a+b+c)^2}.\end{aligned}\]
Suy ra
\[\frac{19(a^2+b^2+c^2)+6(ab+bc+ca)}{a+b+c} \ge 12\sqrt[3]{\frac{3(a^2+b^2+c^2)^2}{a+b+c}}.\]
Ta quy bài toán về chứng minh
\[(a^2+b^2+c^2)^2\ge 9(a+b+c),\]
hay
\[4(a^2+b^2+c^2)^2\ge (a^3+b^3+c^3+abc)(a+b+c).\]
Chuẩn hóa $a+b+c=3$ và đặt $a=x+1,\,b=y+1,\,c=-x-y+1,$ với $xy \ge 0$ bất đẳng thức cần chứng minh trở thành
\[16(x^2+xy+y^2)^2+3(x^2+xy+y^2)+36xy(x+y)\ge0.\]
Theo bất đẳng thức AM-GM thì
\[16(x^2+xy+y^2)^2+3(x^2+xy+y^2) \ge 144x^2y^2+\frac{9}{4}(x+y)^2,\]
lại có
\[144x^2y^2+\frac{9}{4}(x+y)^2+36xy(x+y)=\frac{9(x+y+8xy)^2}{4} \ge 0.\]
Đẳng thức xảy ra khi $a=2b=2c$ cùng các hoán vị. Bài toán được chứng minh.


Bài viết đã được chỉnh sửa nội dung bởi Nguyenhuyen_AG: 26-08-2015 - 10:57

Nguyen Van Huyen
Ho Chi Minh City University Of Transport

#3
Nguyenhuyen_AG

Nguyenhuyen_AG

    Trung úy

  • Thành viên nổi bật 2016
  • 945 Bài viết

4. Bổ đề hoán vị

Trong kỳ thi vô địch toán British MO $1986$ có một bài toán bất đẳng thức thú vị sau:

Bài 4.1. Với $a,\,b,\,c$ là ba số thực thỏa mãn đồng thời các điều kiện
\[a+b+c=0,\,a^2+b^2+c^2=6.\]
Chứng minh rằng
\[{{a}^{2}}b+{{b}^{2}}c+{{c}^{2}}a \le 6.\]

Đây là một bất đẳng thức khó vì đẳng thức của nó không tại tâm, cũng không tại biên mà tại $a=2\cos \frac{2\pi}{9},\,b=2\cos \frac{4\pi}{9},\,c=2\cos \frac{8\pi }{9}$ cùng các hoán vị, chính vì dấu bằng “\textit{kỳ lạ}” này mà bài toán đã gây khó khăn cho các phương pháp mà ta đã biết thậm thậm chí là các phương pháp rất mạnh như S.O.S, dồn biến, ... Trong quyển sách “Sử Dụng Phương Pháp Cauchy Schwarz Để Chứng Minh Bất Đẳng Thức” hai tác giả Võ Quốc Bá Cẩn, Trần Quốc Anh \footnote{Hà Nội} đã đưa ra một chứng minh rất độc đáo bằng bất đẳng thức Cauchy-Schwarz như sau

Lời giải. Ta có một số tính toán đơn giản
\[ab+bc+ca=\frac{(a+b+c)^2-(a^2+b^2+c^2)}{2}=-3,\]
\[a^2b^2+b^2c^2+c^2a^2=(ab+bc+ca)^2-2abc(a+b+c)=9.\]
Sử dụng đẳng thức
\[3({{a}^{2}}b+{{b}^{2}}c+{{c}^{2}}a)=a(2ab+{{c}^{2}})+b(2bc+{{a}^{2}})+c(2ca+{{b}^{2}}),\quad (4.1.1)\]
và bất đẳng thức Cauchy-Schwarz, ta có
\[\begin{aligned}({{a}^{2}}b+{{b}^{2}}c+{{c}^{2}}a)^2 & \le \frac{( {{a}^{2}}+{{b}^{2}}+{{c}^{2}})\left[(2ab+{{c}^{2}})^2+(2bc+{{a}^{2}})^2+(2ca+{{b}^{2}})^2\right]}{9}\\& = \frac{2\left[(2ab+{{c}^{2}})^2+(2bc+{{a}^{2}})^2+(2ca+{{b}^{2}})^2\right]}{3}. \end{aligned}\]
Bằng cách khai triển trực tiếp, ta thấy
\[\sum{{{(2ab+{{c}^{2}})}^{2}}}=(a^2+b^2+c^2)^2+2(a^2b^2+b^2c^2+b^2c^2)+4abc(a+b+c) = 54,\]
nên
\[\left |{{a}^{2}}b+{{b}^{2}}c+{{c}^{2}}a  \right | \le 6,\]
suy ra
\[{{a}^{2}}b+{{b}^{2}}c+{{c}^{2}}a\le 6.\]
Bài toán được chứng minh.

 

Nhận xét. Có thể nói mấu chốt của lời giải này chính là việc sử dụng đẳng thức $(4.1.1)$ để sau đó sử dụng thành công bất đẳng thức Cauchy-Schwarz. Trong sách các tác giả cũng lý giải việc tìm ra đẳng thúc $(4.1.1)$ bằng cách sử dụng phương pháp nhân tử Langrange (sẽ được học trong chương trình toán cao cấp của bậc đại học) như sau. Bằng cách đặt
\[F\left( a,b,c \right)={{a}^{2}}b+{{b}^{2}}c+{{c}^{2}}a+{{\lambda }_{1}}\left( a+b+c \right)+{{\lambda }_{2}}({{a}^{2}}+{{b}^{2}}+{{c}^{2}}-6).\]
Ta thấy điểm cực trị của hàm $F(a,b,c)$ là nghiệm của hệ phương trình sau đây
\[\left\{ \begin{aligned}
  & \frac{\partial F}{\partial a}=\frac{\partial F}{\partial b}=\frac{\partial F}{\partial c}=0 \\
 & a+b+c=0 \\
 & {{a}^{2}}+{{b}^{2}}+{{c}^{2}}=6 \\
\end{aligned} \right.\]
tương đương với
\[\left\{ \begin{aligned}
  & 2ab+{{c}^{2}}+{{\lambda }_{1}}+2{{\lambda }_{2}}a=0 \\
 & 2bc+{{a}^{2}}+{{\lambda }_{1}}+2{{\lambda }_{2}}b=0 \\
 & 2ca+{{b}^{2}}+{{\lambda }_{1}}+2{{\lambda }_{2}}c=0 \\
 & a+b+c=0 \\
 & {{a}^{2}}+{{b}^{2}}+{{c}^{2}}=6 \\
\end{aligned} \right. \quad (4.1.2)\]
Cộng tương ứng theo vế các phương trình thứ nhất, thứ hai, thứ ba lại với nhau ta được
\[{{\left( a+b+c \right)}^{2}}+3{{\lambda }_{1}}+2{{\lambda }_{2}}\left( a+b+c \right)=0,\]
Từ đó suy ra ${{\lambda }_{1}}=0,$ vì thế hệ $(4.1.2)$ lúc này trở thành
\[\left\{ \begin{aligned}
& 2ab+{{c}^{2}}+2{{\lambda }_{2}}a=0 \\
& 2bc+{{a}^{2}}+2{{\lambda }_{2}}b=0 \\
& 2ca+{{b}^{2}}+2{{\lambda }_{2}}c=0 \\
& a+b+c=0 \\
& {{a}^{2}}+{{b}^{2}}+{{c}^{2}}=6 \\
\end{aligned} \right.\]
Từ ba phương trình đầu, ta rút ra được
\[\frac{2ab+{{c}^{2}}}{a}=\frac{2bc+{{a}^{2}}}{b}=\frac{2ca+{{b}^{2}}}{c}=-2{{\lambda }_{2}}.\]
Nếu để ý ta sẽ thấy đẳng thức trên chính là điều kiện xảy ra dấu bằng trong bất đẳng thức Cauchy-Schwarz và điều này lý giải cho việc tách đẳng thức $(4.1.1).$ Không dừng lại ở đây, bằng kỹ thuật tương tự chúng ta ta có thể giải quyết trọn vẹn được bài toán tổng quát của bất đẳng thức trên, một bài toán rất hay.

 

Bài 4.2. Với $a,\,b,\,c$ là ba số thực và $t \ge 0$ là một số cho trước thỏa mãn đồng thời
\[a+b+c=0,\,a^2+b^2+c^2=6t^2.\]
Chứng minh rằng với mọi số thực $k$ ta luôn có bất đẳng thức sau đây
\[\left |{{a}^{2}}b+{{b}^{2}}c+{{c}^{2}}a+kabc  \right |\le 2{{t}^{3}}\sqrt{{{k}^{2}}-3k+9}. \quad (4.2.1)\]

Lời giải. Từ giả thiết ta tính được
\[\begin{aligned} ab+bc+ca & = -3{{t}^{2}}, \\
{{a}^{2}}{{b}^{2}}+{{b}^{2}}{{c}^{2}}+{{c}^{2}}{{a}^{2}} & = 9{{t}^{4}}.\end{aligned}\]
Lại có
\[{{a}^{3}}b+{{b}^{3}}c+{{c}^{3}}a-a{{b}^{3}}-b{{c}^{3}}-c{{a}^{3}}=-(a+b+c)(a-b)(b-c)(c-a)=0,\]
nên ${{a}^{3}}b+{{b}^{3}}c+{{c}^{3}}a=a{{b}^{3}}+b{{c}^{3}}+c{{a}^{3}},$ suy ra
\[\begin{aligned}
{{a}^{3}}b+{{b}^{3}}c+{{c}^{3}}a &= \frac{ab({{a}^{2}}+{{b}^{2}})+bc({{b}^{2}}+{{c}^{2}})+ca({{c}^{2}}+{{a}^{2}})}{2} \\
 & =\frac{ab({{a}^{2}}+{{b}^{2}})+bc({{b}^{2}}+{{c}^{2}})+ca({{c}^{2}}+{{a}^{2}})+abc(a+b+c)}{2} \\
 & =\frac{(ab+bc+ca)({{a}^{2}}+{{b}^{2}}+{{c}^{2}})}{2}=-9{{t}^{4}}.  
\end{aligned}\]
Từ đó áp dụng bất đẳng thức Cauchy-Schwarz, ta có
\[\begin{aligned}{{\left[ 3({{a}^{2}}b+{{b}^{2}}c+{{c}^{2}}a+kabc) \right]}^{2}}&={{\left[ 3({{a}^{2}}b+{{b}^{2}}c+{{c}^{2}}a)+3kabc+k{{t}^{2}}(a+b+c) \right]}^{2}} \\
& ={{\left[ \sum{a(2ab+{{c}^{2}}+kbc+k{{t}^{2}})} \right]}^{2}} \\
& \le ({{a}^{2}}+{{b}^{2}}+{{c}^{2}})\left[ \sum{{{(2ab+{{c}^{2}}+kbc+k{{t}^{2}})}^{2}}} \right]\\&=6t^2\left[ \sum{{{(2ab+{{c}^{2}}+kbc+k{{t}^{2}})}^{2}}} \right].  
\end{aligned}\]
Bằng khai triển trực tiếp ta thấy
\[\sum{{{( 2ab+{{c}^{2}}+kbc+k{{t}^{2}})}^{2}}}=6{{t}^{4}}( {{k}^{2}}-3k+9).\]
Suy ra
\[\left |{{a}^{2}}b+{{b}^{2}}c+{{c}^{2}}a+kabc  \right |\le 2{{t}^{3}}\sqrt{{{k}^{2}}-3k+9}.\]
Bài toán được chứng minh.

 

Nhận xét.

  • Ngoài ra để tính ${{a}^{3}}b+{{b}^{3}}c+{{c}^{3}}a=-9{{t}^{4}}$ ta có thể sử dụng hằng đẳng thức \[(a+b+c)({{a}^{2}}b+{{b}^{2}}c+{{c}^{2}}a)=0.\]
  • Chọn $k=\frac{3}{2}$ thì bất đẳng thức $(4.2.1)$ trở thành

\[\left |2(a^2b+b^2c+c^2a)+3abc  \right |\le 6\sqrt{3}t^3. \quad (4.2.2)\]
Do $a+b+c=0$ nên ta dễ dàng kiểm tra được
\[\left |2(a^2b+b^2c+c^2a)+3abc  \right | = \left |(a-b)(b-c)(c-a)  \right |,\]
khi đó bất đẳng thức $(4.2.1)$ tương đương với
\[\left |(a-b)(b-c)(c-a)  \right |\le 6\sqrt{3}t^3. \quad (4.2.3)\]
Lúc này chọn tiếp $t=1,$ ta sẽ được bài toán rất đẹp sau đây

Với $a,\,b,\,c$ là ba số thực cho trước thỏa mãn đồng thời $a+b+c=0,\,a^2+b^2+c^2=6.$ Chứng minh bất đẳng thức
\[\left |(a-b)(b-c)(c-a)  \right |\le 6\sqrt{3}.\]

(JBMO Turkey 2014)

 

Những bài toán bất đẳng thức khó, hình thức đơn giản, đẹp mắt và có dấu đẳng thức xảy ra khi các biến lệch nhau (lệch tâm và lệch biên) thường không có nhiều, vì để xây dựng được những bài toán như vậy đòi hỏi người ra đề phải có một trình độ lão luyện. Chắc hẳn trong mỗi chúng ta ai cũng đã từng đôi lần chạm trán với những bài toán như vậy. Có thể nêu ra ở đây một bài toán đại diện cho những tiêu chuẩn nói trên, một bất đẳng thức rất nổi tiếng của giáo sư Vasile Cirtoaje.

 

Bài 4.3. Cho ba số thực $a,\,b,\,c$ thay đổi bất kỳ. Chứng minh rằng
\[(a^2+b^2+c^2)^2 \ge 3(a^3b+b^3c+c^3a).\]

 

Ẩn bên trong vẻ bề ngoài đơn giản này là một bất đẳng thức vô cùng khó vì ngoài trường hợp tầm thường $a=b=c$ để đẳng thức xảy ra thì vẫn còn một trường hợp nữa đặc biệt nữa là $(a,\,b,\,c) \sim \left(\sin^2\frac{\pi}{7},\,\sin^2\frac{2\pi}{7},\,\sin^2\frac{4\pi}{7}\right),$ chính vì thế mà một lời giải bình thường cho bất đẳng thức có dấu bằng “\textit{bất thường}” này dường như là không có ! Những chứng minh ban đầu đa phần đều đưa bài toán về dạng tổng các bình phương (ngay cả bằng phương pháp tam thức bậc hai cũng buộc ta phải phân tích biệt thức $\Delta$ thành một tổng bình phương) điều này đòi hỏi người làm toán phải có một nhãn quan nhạy bén để nhận biết được sự tồn tại của các đại lượng bình phương đó. Bằng bài toán tổng quát trên chúng ta có một lời giải khá thú vị sau đây.

Lời giải. Tương tự bài $(2.9)$ ta chuẩn hóa $a+b+c=3,$ khi đó tồn tại số thực $t \geqslant 0$ sao cho
\[{{a}^{2}}+{{b}^{2}}+{{c}^{2}}=3+6{{t}^{2}}.\]
Tiếp đến đặt $a=1+x,\, b=1+y,\, c=1+z$ thì $x+y+z=0$ và tính được
$${{x}^{2}}+{{y}^{2}}+{{z}^{2}}=6{{t}^{2}}.$$
Bất đẳng thức cần chứng minh trở thành
\[3{{(1+2{{t}^{2}})}^{2}}\ge {{a}^{3}}b+{{b}^{3}}c+{{c}^{3}}a.\]
Trong (4.2.1) chọn $k=1,$ với chú ý $\left |A  \right | \ge A,$ ta được
\[{{a}^{2}}b+{{b}^{2}}c+{{c}^{2}}a+abc\le 2\sqrt{7}\cdot{{t}^{3}}.\]
Biến đổi vế trái
\[\begin{aligned} {{a}^{3}}b+{{b}^{3}}c+{{c}^{3}}a&={{(1+x)}^{3}}(1+y)+{{(1+y)}^{3}}(1+z)+{{(1+z)}^{3}}(1+x) \\& =\sum{({{x}^{3}}+3{{x}^{2}}y+3xy+{{x}^{3}}y+3{{x}^{2}}+1)} \\
 & ={{x}^{3}}+{{y}^{3}}+{{z}^{3}}+3({{x}^{2}}y+{{y}^{2}}z+{{z}^{2}}x)+3\left( \sum{{{x}^{2}}}+\sum{yz} \right)+\sum{{{x}^{3}}y}+3 \\& =3xyz+3({{x}^{2}}y+{{y}^{2}}z+{{z}^{2}}x)-9{{t}^{4}}+9{{t}^{2}}+3 \\& = 3({{x}^{2}}y+{{y}^{2}}z+{{z}^{2}}x+xyz)-9{{t}^{4}}+9{{t}^{2}}+3 \\& \le 3\cdot 2\sqrt{7}\cdot{{t}^{3}}-9{{t}^{4}}+9{{t}^{2}}+3. \end{aligned}\]
Như vậy ta chỉ cần chứng minh  
\[{{(1+2{{t}^{2}})}^{2}}\ge 2{{t}^{3}}\sqrt{7}-3{{t}^{4}}+3{{t}^{2}}+1,\]
hay là
\[7{{t}^{4}}+{{t}^{2}}\ge 2\sqrt{7}{{t}^{3}}.\]
Áp dụng bất đẳng thức AM-GM ta có
\[7{{t}^{4}}+{{t}^{2}}\ge 2\sqrt{7t^4\cdot t^2} = 2\sqrt{7}{{t}^{3}}.\]
Bài toán được chứng minh.

 

Bài 4.4. Tìm hằng số $M$ nhỏ nhất sao cho bất đẳng thức
\[\left | ab(a^2-b^2)+bc(b^2-c^2)+ca(c^2-a^2) \right | \le M(a^2+b^2+c^2)^2,\]
luôn đúng với mọi số thực $a,\,b,\,c$ bất kỳ.

(IMO 2006)

Lời giải. Cho $a = \frac{3\sqrt{2}+2}{2}, \,b = \frac{2-3\sqrt{2}}{2},\,c = 1$ thì $M \ge \frac{9\sqrt{2}}{32},$ ta sẽ chứng minh $M = \frac{9\sqrt{2}}{32}$ là giá trị nhỏ nhất cần tìm, tức chứng minh
\[\left | ab(a^2-b^2)+bc(b^2-c^2)+ca(c^2-a^2) \right | \le \frac{9\sqrt{2}}{32}(a^2+b^2+c^2)^2.\]
 Thật vậy bất đẳng thức tương đương với
\[\left | (a+b+c)(a-b)(b-c)(c-a) \right | \le \frac{9\sqrt{2}}{32}(a^2+b^2+c^2)^2. \quad (4.4.1)\]
Chuẩn hóa $a+b+c=3,$ khi đó tồn tại số thực $t \ge 0$ để $a^2+b^2+c^2=3+6t^2,$ lúc này $(4.4.1)$ được viết lại dưới dạng
\[\left |(a-b)(b-c)(c-a) \right | \le \frac{27\sqrt{2}}{32}(1+2t^2)^2. \quad (4.4.2)\]
Đặt $a=x+1,\,b=y+1,\,c=z+1$ rồi thay vào (4.4.2) ta được
\[\left |(x-y)(y-z)(z-x) \right | \le \frac{27\sqrt{2}}{32}(1+2t^2)^2.\]
Áp dụng bất đẳng thức $(4.2.3)$ ta đưa bài toán về chứng minh bất đẳng thức mạnh hơn là

\[6\sqrt{3}\cdot{t^3} \le \frac{27\sqrt{2}}{32}(1+2t^2)^2,\]
hay
\[(1+2t^2)^2 \ge \frac{32\sqrt{6}}{9}\cdot{t^3}.\]
Áp dụng bất đẳng thức AM-GM, ta có
\[1+2t^2 = 1 + \frac{2}{3}\cdot{t^2} + \frac{2}{3}\cdot{t^2} + \frac{2}{3}\cdot{t^2} \ge 4\sqrt[4]{1\cdot\left ( \frac{2}{3}\cdot{t^2} \right )^3}=4\sqrt[4]{\frac{8}{27}\cdot{t^6}},\]
bình phương hai vế ta được
\[(1+2t^2)^2  \ge \frac{32\sqrt{6}}{9}\cdot {t^3}.\]
Điều này cho phép ta kết luận $M = \frac{9\sqrt{2}}{32},$ là giá trị nhỏ nhất cần tìm.

 

Bài 4.5. Với $a,\,b,\,c$ là ba số thực thỏa mãn đồng thời các điều kiện
\[a+b+c=0,\,a^2+b^2+c^2=3.\]
Chứng minh rằng
\[a^5b+b^5c+c^5a \le -3.\]

Lời giải Do $a^2+b^2+c^2=3$ nên từ bài $(4.2)$ ta được $t=\frac{1}{\sqrt{2}}.$ Ta có có một số tính toán
\[\begin{aligned}ab+bc+ca & = -\frac{3}{2}, \\ a^3b+b^3c+c^3a & = -\frac{9}{4}. \\ a^2b+b^2c+c^2a+3abc &= - (ab^2+bc^2+ca^2) \end{aligned}\]

\[a^3b^3+b^3c^3+c^3a^3=(ab+bc+ca)^3+3a^2b^2c^2=-\frac{27}{8}+3a^2b^2c^2.\]
Ta có biến đổi
\[a^5b=a^3b\cdot{a^2}=a^3b(3-b^2-c^2)=3a^3b-a^3b^3-a^3bc^2,\]
suy ra
\[\begin{aligned} a^5b+b^5c+c^5a &= 3(a^3b+b^3c+c^3a)-(a^3b^3+b^3c^3+c^3a^3)-(ab^2+bc^2+ca^2)abc \\& =3\cdot\left(-\frac{9}{4}\right)-\left ( -\frac{27}{8}+3a^2b^2c^2 \right)+(a^2b+b^2c+c^2a+3abc)abc \\& =-\frac{27}{8} +abc(a^2b+b^2c+c^2a).\end{aligned}\]
Bài toán quy về chứng minh
\[abc(a^2b+b^2c+c^2a) \le \frac{3}{8}.\]
Trong (4.2.1) chọn $k=3,\,t=\frac{1}{\sqrt{2}},$ ta được
\[\left |a^2b+b^2c+c^2a+3abc  \right |\le \frac{3}{\sqrt{2}}. \quad (4.6.1)\]
Áp dụng bất đẳng thức AM-GM, ta có
\[\begin{aligned} abc(a^2b+b^2c+c^2a) &= \frac{1}{3}\cdot 3abc\cdot (a^2b+b^2c+c^2a) \\& \le \frac{1}{3}\left ( \frac{3abc+a^2b+b^2c+c^2a}{2} \right )^2\\& \le \frac{3}{8}.\end{aligned}\]
Đẳng thức xảy ra khi và chỉ khi $(a,b,c)=\left (\sqrt{2}\cos\dfrac{2\pi}{9},\,\sqrt{2}\cos\dfrac{13\pi}{9},\,\sqrt{2}\cos \dfrac{7\pi}{9}  \right ),$ hoặc $(a,b,c)=\left (-\sqrt{2}\cos\dfrac{2\pi}{9},\,-\sqrt{2}\cos\dfrac{13\pi}{9},\,-\sqrt{2}\cos\dfrac{7\pi}{9}  \right )$ cùng các hoán vị.

Bài toán được chứng minh.


Nhận xét. Chắc hẳn bạn đọc sẽ thắc mắc tại sao ta lại chọn $k=3$ để có đánh giá $(4.6.1).$ Ta có thể lý giải điều này như sau

Do $t=\frac{1}{\sqrt{2}},$ nên từ $(4.2.1)$ ta được
\[\left |{{a}^{2}}b+{{b}^{2}}c+{{c}^{2}}a+kabc  \right |\le \sqrt{\frac{k^2-3k+9}{2}}.\]
Áp dụng bất đẳng thức AM-GM ta có
\[\begin{aligned} abc(a^2b+b^2c+c^2a) &\le \left |\frac{1}{k}\right | \cdot \left |kabc\cdot (a^2b+b^2c+c^2a)  \right | \\& \le \left |\frac{1}{k}  \right |\left ( \frac{kabc+a^2b+b^2c+c^2a}{2} \right )^2\\& \le \frac{k^2-3k+9}{8\left |k  \right |}.\end{aligned}\]
Như vậy ta cần tìm $k$ sao cho
\[k^2-3k+9 = 3\left |k\right |.\]
Giải phương trình này ta được $k=3.$


Nguyen Van Huyen
Ho Chi Minh City University Of Transport

#4
Nguyenhuyen_AG

Nguyenhuyen_AG

    Trung úy

  • Thành viên nổi bật 2016
  • 945 Bài viết

5. Bài tập rèn luyện

Bài 5.1. Cho $a, \, b, \, c$ là ba số thực không âm sao cho $ab+bc+ca>0.$ Chứng minh rằng
\[1 \leqslant \frac{a^2}{2a^2+(b+c)^2}+\frac{b^2}{2b^2+(c+a)^2}+\frac{c^2}{2c^2+(a+b)^2} \le \frac{2}{3}.\]
Bài 5.2. Chứng minh rằng bất đẳng thức
\[7\left[a^2+b^2+c^2+(a+b+c)^2\right]^2 \ge 12\left[a^4+b^4+c^4+(a+b+c)^4\right],\]
luôn đúng với mọi số thực $a, \, b, \, c$ thay đổi bất kỳ.

Bài 5.3. Cho $a, \, b, \, c$ là các số thực không âm.Chứng minh rằng
\[a^4+b^4+c^4-abc(a+b+c) \geqslant 2\sqrt{2}\left|a^3b+b^3c+c^3a-ab^3-bc^3-ca^3\right|.\]
Bài 5.4. Cho ba số thực $x,\,y,\,z$ có tổng bằng $3.$ Chứng minh rằng
\[4(x^4+y^4+z^4)+45 \ge 19(x^2+y^2+z^2).\]
Bài 5.5. Cho $a,\,b,\,c$ là độ dài ba cạnh của tam giác. Chứng minh rằng
\[a^4+b^4+c^4+9abc(a+b+c) \le 10(a^2b^2+b^2c^2+c^2a^2).\]
Bài 5.6. Cho $a,\,b,\,c$ là ba số thực thỏa mãn $a^2+b^2+c^2=3.$ Tìm giá trị lớn nhất của
\[P = a^4+b^4+c^4+3(ab+bc+ca).\]
Bài 5.7. Cho $a,\,b,\,c$ là ba số thực thỏa mãn $ab+bc+ca=3.$ Tìm giá trị nhỏ nhất của
\[F = 4(a^4+b^4+c^4)+11abc(a+b+c).\]
Bài 5.8. Tìm hằng số $k$ lớn nhất sao cho bất đẳng
\[a^4+b^4+c^4 \ge 3 + k(a^3+b^3+c^3+3abc-6),\]
luôn đúng với mọi số thực dương $a,\,b,\,c$ thỏa mãn $a+b+c=3.$

Bài 5.9. Chứng minh rằng với mọi số thực không âm $a,\,b,\,c$ thì
\[a^4+b^4+c^4-a^2b^2-b^2c^2-c^2a^2 \geqslant 2 \left|a^3b+b^3c+c^3a-ab^3-bc^3-ca^3\right|.\]

Bài 5.10. Cho ba số thực $\alpha,\,\beta,\,\gamma$ thỏa mãn $1+\alpha+\beta=2\gamma.$ Chứng minh rằng bất đẳng thức
\[\sum a^4 + \alpha \sum b^2c^2 +\beta abc \sum a \ge \gamma \sum bc(b^2+c^2),\]
luôn đúng với mọi số thực $a,\,b,\,c$ khi và chỉ khi $1+\alpha \ge \gamma^2.$


6. Tài liệu tham khảo

  • Võ Quốc Bá Cẩn, Chuyên Đề Bất Đẳng Thức Hiện Đại, 2008.

 

  • Võ Quốc Bá Cẩn, Trần Quốc Anh, Sử Dụng Phương Pháp Cauchy Schwarz Để Chứng Minh Bất Đẳng Thức, Nhà Xuất Bản Đại Học Sư Phạm, 2010.

 

  • Lê Việt Hải, Phương Pháp Nhân Tử Langrange & Bất Đẳng Thức Cauchy-Schwarz, Câu Lạc Bộ Toán Trường Phổ Thông Năng Khiếu Thành Phố Hồ Chí Minh, 2011.

 

  • Vasile Cirtoaje, Algebraic Inequalities Polynomial Rational Symmetric Inequalities, GIL Publishing House, 2011.

 

  • Vasile Cirtoaje, Võ Quốc Bá Cẩn , On Some Cyclic Homogeneous Polynomial Inequality Of Degree Fourth In Real Variables Under Constraints, International Journal of Pure and Applied Mathematics, 2012.

 


Nguyen Van Huyen
Ho Chi Minh City University Of Transport

#5
Nguyenhuyen_AG

Nguyenhuyen_AG

    Trung úy

  • Thành viên nổi bật 2016
  • 945 Bài viết

Đây là file pdf của bài viết.

File gửi kèm


Bài viết đã được chỉnh sửa nội dung bởi Nguyenhuyen_AG: 26-08-2015 - 11:24

Nguyen Van Huyen
Ho Chi Minh City University Of Transport

#6
Nguyenhuyen_AG

Nguyenhuyen_AG

    Trung úy

  • Thành viên nổi bật 2016
  • 945 Bài viết
Mọi người đọc có sai xót hay cần chỉnh sửa gì không. Cho hai tác giả ý kiến. :D
Nguyen Van Huyen
Ho Chi Minh City University Of Transport

#7
mathstu

mathstu

    Hạ sĩ

  • Thành viên
  • 51 Bài viết

dạ cái này quả thật rất hay, lúc cầm cuốn GGTH đập vào mặt cái bài này ùi nên thấy rất mê, em cứ tưởng là bài viết của thầy CẨn nữa chứ, 


Bài viết đã được chỉnh sửa nội dung bởi mathstu: 29-08-2015 - 22:31

Họ cười tôi vì tôi khác họ    

             

             Tôi cười họ vì tôi mắc cười    >:)  >:)  >:) 


#8
locnguyen2207

locnguyen2207

    Trung sĩ

  • Thành viên
  • 102 Bài viết

Quả thật rất hay......cảm ơn nhiều nhiều


                 hinh-dong-hai-huoc-23.gif


#9
Element hero Neos

Element hero Neos

    Trung úy

  • Thành viên
  • 943 Bài viết

có giải được phương trình bậc 4 bằng phương pháp hàm số hay miền giá trị được không



#10
Element hero Neos

Element hero Neos

    Trung úy

  • Thành viên
  • 943 Bài viết

Đây là file pdf của bài viết.

sao mn ko mở được



#11
Nguyenhuyen_AG

Nguyenhuyen_AG

    Trung úy

  • Thành viên nổi bật 2016
  • 945 Bài viết

sao mn ko mở được

 

Mình vẫn tải được file và mở lên bình thường mà.


Nguyen Van Huyen
Ho Chi Minh City University Of Transport

#12
Element hero Neos

Element hero Neos

    Trung úy

  • Thành viên
  • 943 Bài viết

Mình vẫn tải được file và mở lên bình thường mà.

ờ, được rồi



#13
Element hero Neos

Element hero Neos

    Trung úy

  • Thành viên
  • 943 Bài viết

dạ cái này quả thật rất hay, lúc cầm cuốn GGTH đập vào mặt cái bài này ùi nên thấy rất mê, em cứ tưởng là bài viết của thầy CẨn nữa chứ, 

Lạc đề nha bạn!






2 người đang xem chủ đề

0 thành viên, 2 khách, 0 thành viên ẩn danh